Középiskolai Matematikai és Fizikai Lapok
Informatika rovattal
Kiadja a MATFUND Alapítvány
Már regisztráltál?
Új vendég vagy?

Fórum: Valaki mondja meg!

  [1]    [2]    [3]    [4]    [5]    [6]    [7]    [8]    [9]    [10]    [11]    [12]    [13]    [14]    [15]    [16]    [17]    [18]    [19]    [20]    [21]    [22]    [23]    [24]    [25]    [26]    [27]    [28]    [29]    [30]    [31]    [32]    [33]    [34]    [35]    [36]    [37]    [38]    [39]    [40]    [41]    [42]    [43]    [44]    [45]    [46]  

Szeretnél hozzászólni? Jelentkezz be.
[2287] marcius82023-02-16 21:59:33

Először is köszönöm Johnny 10 és Sakkhmath alapvetően segítő jellegű válaszait.

Én csak egy egyszerű határértékre kérdeztem, ami nem volt sürgős, március közepéig ráért. Persze azóta kaptam máshonnan segítséget, Johnny 10 hivatkozott egy Kömal-feladatra. Megnéztem én is az említett feladatot. Ott egy diofantoszi egyenletet kellett megoldani. Ha az egyenletet elosztom a jobb oldallal, akkor a jobb oldalon 1 van, a bal oldalon egy az, általam kérdezett sorozathoz hasonló jellegű sorozat van. Ha erről a hasonló jellegű sorozatról megmutatom, hogy konvergens, van határértréke, azzal még nem oldom meg a diofantoszi egyenletet. Ugyanis bármennyire is konvergens az a hasonló jellegű sorozat, még lehet nagyon sok (nem végtelen) olyan "kiugró" tagja, amelynek értéke mondjuk pl 1. Hogy tudjam használni a hasonló jellegű sorozat konvergenciáját, ahhoz az kell, hogy minden \(\displaystyle \epsilon>0\) értékhez találjak egy \(\displaystyle n\) értéket, ahonnan kezdve a sorozat minden tagjának a határértéktől vett eltérése \(\displaystyle \epsilon\)-nál kisebb legyen. Ez viszont egy újabb feladat, amit most nagy hirtelen nem tudnék megcsinálni. Akkor már jobb, ha azt mutatom meg a hasonló jellegű sorozatról, hogy szigorúan monoton csökkenő.

Mindenfajta személyeskedést elkerülve nem volt szándékom az említett versenyfeladat megoldására vagy annak egy részére rákérdezni, ezúton is bocsánatot kérek a szerkesztőségtől, a versenyzőktől és a versenybizottságtól. Tisztelettel: BZ.

Előzmény: [2282] Johnny 10, 2023-02-14 19:17:19
[2288] marcius82023-02-16 22:28:25

Nekem így lett meg a határérték. Mondjuk nem túl precíz.....

\(\displaystyle \lim_{n\to\infty}\frac{1^n+2^n+3^n+\dots" +(n-2)^n+(n-1)^n+n^n}{(n+1)^n}=\)

\(\displaystyle \lim_{n\to\infty}\frac{n^n+(n-1)^n+(n-2)^n+(n-3)^n+\dots" +3^n+2^n+1^n}{(n+1)^n}=\)

\(\displaystyle \lim_{n\to\infty}\Bigg(\Bigg(\frac{n}{n+1}\Bigg)^n+\Bigg(\frac{n-1}{n+1}\Bigg)^n+\Bigg(\frac{n-2}{n+1}\Bigg)^n+\Bigg(\frac{n-3}{n+1}\Bigg)^n+\dots\Bigg)=\)

\(\displaystyle e^{-1}+e^{-2}+e^{-3}+e^{-4}+\dots\)=végtelen mértani sor=

\(\displaystyle \frac{e^{-1}}{1-e^{-1}}\)=\(\displaystyle \frac{1}{e-1}\)

Menet közben felhasználtam a következő határértéket:

\(\displaystyle \lim_{n\to\infty}\Bigg(\Bigg(\frac{n+a}{n+b}\Bigg)^n\Bigg)=e^{a-b}\)

[2289] nadorp2023-02-17 14:37:55

Az alábbi rész kérdőjeles:

\(\displaystyle \lim_{n\to\infty}\Bigg(\frac{n}{n+1}\Bigg)^n+\Bigg(\frac{n-1}{n+1}\Bigg)^n+\Bigg(\frac{n-2}{n+1}\Bigg)^n+\Bigg(\frac{n-3}{n+1}\Bigg)^n+\dots ?= e^{-1}+e^{-2}+e^{-3}+e^{-4}+\dots\)

Ha \(\displaystyle b_n=\Bigg(\frac{n}{n+1}\Bigg)^n+\Bigg(\frac{n-1}{n+1}\Bigg)^n+\dots+ \Bigg(\frac{1}{n+1}\Bigg)^n\)

akkor attól még, hogy a tagok "függőlegesen" konvergálnak, tehát hogy fix k-ra

\(\displaystyle \lim_{n\to\infty}\Bigg(\frac{n-k}{n+1}\Bigg)^n=e^{-k-1}\)

még nem következik, hogy az \(\displaystyle e^{-k-1}\) határértékek összege megegyezik \(\displaystyle b_n\) határértékével. Az még elmondható, hogy \(\displaystyle b_n\) egy felső korlátja a mértani sor - bár már ehhez is hozzá kéne tenni , hogy ez a fenti limesz monotonitása miatt van - de hogy oda konvergál-e, az nem következik az általad leírtakból.

Ha \(\displaystyle b_{n,k}=\Bigg(\frac{n-k}{n+1}\Bigg)^n\) ha \(\displaystyle 0\leq k \leq n\) és \(\displaystyle b_{n,k}=0\) ha \(\displaystyle k>n\), akkor azt állítod, hogy

\(\displaystyle \lim_{n\to\infty}\sum_{k=0}^nb_{n,k}=\lim_{n\to\infty}\sum_{k=0}^\infty b_{n,k}=\sum_{k=0}^\infty(\lim_{n\to\infty}b_{n,k})\)

Viszont a fenti határátmenet csere nem nyilvánvaló.

A "szabályos" eljárás az, ami sakkmath 2. megoldásában van (Tannery-tétel). A képletek (indexek) egyszerűbbek lesznek, ha sakkmath megoldását követve a \(\displaystyle \lim_{n\to\infty}\sum_{k=1}^{n}\left(\frac{k}{n}\right)^n=\lim_{n\to\infty}S_n\) határértékét számoljuk először ki, aztán osztjuk e-vel.

Minden \(\displaystyle n\geq1\) esetén legyen:

\(\displaystyle a_{n,k}=\Bigg\{\begin{matrix}\bigg(\frac{n-k}{n}\bigg)^n & 0\leq k\leq n \\ 0 & k> n \\ \end{matrix}\).

Ez egy végtelen mátrix, aminek a főátlójában - kivéve az \(\displaystyle a_{1,0}\) -és felette minden elem 0, a nulladik (azaz az első) oszlop csupa 1-es.

Bármely fix k-ra ismert, hogy az \(\displaystyle a_{n,k}\) sorozat monoton növekvő és határértékére

\(\displaystyle \lim_{n\to\infty}a_{n,k}=e^{-k} \tag{1}\)

Legyen \(\displaystyle \varepsilon>0\) rögzített. Mivel \(\displaystyle \sum_{k=0}^{\infty}e^{-k}=\frac{e}{e-1}=S\), ezért létezik K, hogy

\(\displaystyle \sum_{k=0}^{K}e^{-k}>S-\varepsilon \tag{2}\)

Másrészt (1)-ben a monotonitás miatt

létezik \(\displaystyle N_0\), hogy \(\displaystyle n>N_0\) esetén \(\displaystyle 1-a_{n,0}<\frac\varepsilon {K+1}\)

létezik \(\displaystyle N_1\), hogy \(\displaystyle n>N_1\) esetén \(\displaystyle \frac1{e}-a_{n,1}<\frac\varepsilon {K+1}\)

...

létezik \(\displaystyle N_K\), hogy \(\displaystyle n>N_K\) esetén \(\displaystyle \frac1{e^K}-a_{n,K}<\frac\varepsilon {K+1}\)

Így, ha \(\displaystyle N:=\max(K,N_0,...,N_K)\), akkor összeadva a fenti K+1 darab egyenlőtlenséget, \(\displaystyle n>N\) esetén teljesül

\(\displaystyle \sum_{k=0}^{K}e^{-k}-\sum_{k=0}^{K}a_{n,k}<\varepsilon\)

Felhasználva (2)-t is:

\(\displaystyle \varepsilon>\sum_{k=0}^{K}e^{-k}-\sum_{k=0}^{K}a_{n,k}> S-\varepsilon-\left(\sum_{k=0}^{n}a_{n,k}-\sum_{k=K+1}^{n}a_{n,k}\right)>S-\varepsilon-\sum_{k=0}^{n}a_{n,k}=S-S_n-\varepsilon\)

Azaz \(\displaystyle S-S_n<2\varepsilon\)

Előzmény: [2288] marcius8, 2023-02-16 22:28:25
[2290] nadorp2023-02-17 16:59:29

Elnézést kérek, de az előző hozzászólásban alábbi mondat nem igaz:

Az még elmondható, hogy \(\displaystyle b_n\) egy felső korlátja a mértani sor - bár már ehhez is hozzá kéne tenni , hogy ez a fenti limesz monotonitása miatt van

A \(\displaystyle b_{n,k}\) sorozat ui. csak elég nagy n-re kisebb a határértéknél, de ez nem nyilvánvaló állítás. Ezért is szerencsésebb a megoldásban tekintett másik sorozat, \(\displaystyle a_{n,k}\) vizsgálata, mert az monoton nő.

Előzmény: [2289] nadorp, 2023-02-17 14:37:55
[2291] marcius82023-02-21 09:52:55

igen, erre mondtam, hogy nem túl precíz...

Előzmény: [2288] marcius8, 2023-02-16 22:28:25
[2292] marcius82023-04-26 21:17:36

Megint találkoztam egy feladattal: Bizonyítandó, hogy a síkon bárhogy is választunk ki négy pontot, a köztük előforduló hat távlság maximumának és minimumának hányadosa legalább \(\displaystyle 2^{1/2}\). Inkább arról van szó, hogy hogyan kell szépen és egyszerűen bizonyítani, mert körülményesen össze tudom hozni a bizonyítást. Minden segítséget előre is köszönök.

[2293] sakkmath2023-04-27 15:48:51

A bolyai.hu honlapról könnyű eljutni a [Kürschák József Matematikai Tanulóverseny 1961 1. kateg. 1. ford. 9-13. évfolyam] mappáig. Innen másolom be két részletben Hajós György (!) 1. feladatra adott megoldását. E feladat eredeti kitűzési szövege:

A sík négy pontja hat távolságot határoz meg. Bizonyítsuk be, hogy e távolságok legnagyobbika a legkisebbel osztva nem adhat \(\displaystyle \sqrt2\) -nél kisebb hányadost.

A megoldás első része:

Előzmény: [2292] marcius8, 2023-04-26 21:17:36
[2294] sakkmath2023-04-27 15:54:26

A megoldás második része:

Előzmény: [2293] sakkmath, 2023-04-27 15:48:51
[2295] marcius82023-04-27 20:23:13

köszi szépen a szépmegfogalmazást. Valahogy én is így gondoltam, csak nem tudtam ennyire egyszerűen és átláthatóan összefoglalni.

Előzmény: [2294] sakkmath, 2023-04-27 15:54:26
[2296] BerkoErzsebet2023-04-28 05:57:12

Egy könyvemből (Matematikai versenytételek) másolok.

II. megoldás: Ha a derékszögű háromszög átfogóját a kisebbik (pontosabban: a másiknál nem nagyobb) befogóval elosztjuk, gyök2-nél nem kisebb hányadoshoz jutunk. Ez abból következik, hogy a<= b esetén c*c=a*a+b*b>=2*a*a, tehát c/a>=gyök2.

Ha a háromszög két oldalát változatlanul hagyjuk, de az általuk közrefogott szöget növeljük, akkor a harmadik oldal is növekszik. Ezért tompaszögű háromszögre, sőt (egy egyenesen elhelyezkedő, csatlakozó szakaszokká) elfajuló háromszögre is kimondhatjuk, hogy legnagyobb oldala a legkisebbel osztva legalább gyök2 értékű hányadost ad.

Elég ezért azt bizonyítanunk, hogy a sík bármely négy pontja között van három olyan, amely derékszögű, tompaszögű vagy elfajuló háromszöget határoz meg. Induljunk ki a sík négy pontjából. Feltehetjük, hogy nincs közöttük három egy egyenesen elhelyezkedő. Tekintsük a négy pont konvex burkát, azaz azt az idomot, amelyet úgy kapunk, hogy a pontok köré fonalat feszítünk. Minthogy a pontok nincsenek mindannyian egy egyenesen, konvex burkuk vagy háromszög, vagy négyszög. Ha a P1P2P3 háromszöghöz jutunk, akkor P4 ennek belsejében van, hiszen három pont nem lehet egy egyenesen. A P4P1, P4P2, P4P3 szakaszok háromszögünket három háromszögre vágják fel. Ezeknek P4-nél elhelyezkedő három szöge együttesen 360 fok, s ezért közöttük tompaszög is van (sőt közülük legalább kettő tompa.) Egy ilyen tompaszög a pontjainkból alakított tompaszögű háromszög szöge. Ha a konvex burok négyszög, akkor ennek szögeiről elmondhatjuk, hogy nem lehet mindegyik hegyes, hiszen összegük 360 fok. A legnagyobb szög tehát derékszög vagy tompaszög, s ez egy a pontjainkból alakított derékszögű vagy tompaszögű háromszög szöge.

[2297] marcius82023-04-28 09:20:57

Köszönöm a szép megfogalmazásokat. Ilyenkor látom, hogy bár ha tudom a feladat megoldását, vagy legalábbis azt hiszem, hogy tudom a feladat megoldását, még nem biztos, hogy jól le tudom írni. Találkoztam további két feladattal, amelyet tudom, hogy igaz, azt is tudom, hogy miért, de nem tudom szépen megfogalmazni:

1. Adott térben véges sok pont úgy, hogy semelyik három nincs egy egyenesen. Igaz-e, hogy ha bármelyik négy pont illeszkedik egy síkra, akkor az összes pont egy síkon van? Mi a helyzet végtelen sok pont esetén.

2. Igaz-e, hogy egy \(\displaystyle n\) oldalú konvex sokszögbe maximum \(\displaystyle n-3\) átlót lehet behúzni, úgy hogy az átlóknak ne legyen metszéspontjuk?

[2298] Lpont2023-04-28 13:26:55

Az 1. feladatra az alábbi - síkbeli - Sylvester-Gallai tétel bizonyításának gondolatmenetét lehet felhasználni a térben is.

Sylvester-Gallai tétel: Adott n>=3 pont a síkon. Ekkor vagy az összes adott pont illeszkedik egy egyenesre, vagy van egy olyan egyenes, amire közülük pontosan kettő illeszkedik!

A tétel bizonyítása az interneten is elérhető több oldalon is.

Előzmény: [2297] marcius8, 2023-04-28 09:20:57
[2299] nadorp2023-04-29 14:06:04

Bizonyítás teljes indukcióval.

Ha n=3, akkor igaz az állítás, mert ekkor n-3=0 és a háromszögnek nincs átlója.

Ha n=4, igaz az állítás, mert ekkor n-3=1 és mivel egy konvex négyszög 2 átlója metszi egymást, ezért csak egy átlót húzhatunk be.

Legyen \(\displaystyle n\geq5\), \(\displaystyle K_n\) egy n oldalú konvex sokszög és tegyük fel, hogy n-nél kisebb oldalszámú konvex sokszögre igaz az állítás.

Húzzunk be egy tetszőleges \(\displaystyle f\) átlót \(\displaystyle K_n\)-ben. Ekkor a sokszöget egy i oldalú \(\displaystyle K_i\) és egy n+2-i oldalú \(\displaystyle K_{n+2-i}\) konvex sokszögre bontottuk fel \(\displaystyle (3\leq i\leq n-1\)). Nyilvánvaló, hogy ezek után csak olyan átlót húzhatunk be \(\displaystyle K_n\)-ben, mely vagy csak \(\displaystyle K_i\) vagy csak \(\displaystyle K_{n+2-i}\) belsejében halad, ugyanis ellenkező esetben a behúzott átló metszené \(\displaystyle f\)-et. Az indukciós feltevés miatt így legfeljebb összesen

\(\displaystyle i-3+n+2-i-3=n-4\) átlót húzhattunk be.

Ezekhez hozzávéve a \(\displaystyle f\) átlót, kapjuk, hogy \(\displaystyle K_n\)-be is legfeljebb n-3 egymást nem metsző átló húzható be,mely tartalmazza \(\displaystyle f\)-et. A kapott felső érték nem függ i-től (így \(\displaystyle f\)-től sem), tehát \(\displaystyle K_n\)-be is legfeljebb n-3 darab egymást nem metsző átló húzható be.

A kapott felső korlát mindig elérhető, ha egy kijelölt fix csúcsból húzzuk be az összes átlót.

Előzmény: [2297] marcius8, 2023-04-28 09:20:57
[2300] nadorp2023-04-30 07:58:43

Szerintem a 1. feladatra a Sylvester-Gallai tétel túl erős, csak akkor kéne, ha azt a gyengébb feltételt vesszük, hogy bármely 3 pont által kifeszített síkra illeszkedik egy tőlük különböző negyedik pont.

Az 1. feladatban válasszunk tetszőleges A,B,C pontokat. Ezek nem kollineárisak, tehát egyértelműen meghatároznak egy S síkot. Ha most veszünk egy, az előző pontoktól különböző P pontot, akkor a feltétel szerint A,B,C és P egy síkon helyezkednek el, de ez a sík tartalmazza az A,B,C pontokat is, tehát meg kell hogy egyezzen S-sel. Tehát a pontok egy síkon vannak.

Előzmény: [2298] Lpont, 2023-04-28 13:26:55
[2301] marcius82023-06-26 20:14:02

Van 20 darab egymástól nem feltétlenül különböző szám. A 20 darab számot tetszőlegesen szétválasztjuk két 10 elemű halmazra. Ezután egy 2 sorból és 10 oszlopból álló táblázat felső sorába beírjuk az egyik 10 elemű halmaz elemeit növekvő (nem csökkenő) sorrendben, a táblázat alsó sorába beírjuk a másik 10 elemű halmaz elemeit csökkenő (nem növekvő) sorrendben Ezután a táblázat minden oszlopában kiszámoljuk az oszlopban levő két szám különbségét (különbség = nagyobb szám - kisebb szám), majd az így kapott különbségeket összeadjuk, így egy eredmény adódik. Bizonyítsuk be, hogy bárhogy is választjuk szét a 20 darab számot két 10 elemű halmazra, a kapott eredmény mindig kiszámolható úgy is, hogy a 10 legnagyobb szám összegéből kivonjuk a 10 legkisebb szám összegét. Mindenki segítségét előre is köszönöm: BZ.

[2302] marcius82023-06-26 23:04:52

ok, azóta megtaláltam a választ...

Előzmény: [2301] marcius8, 2023-06-26 20:14:02
[2303] marcius82023-07-19 18:45:26

Egy matekos kérdésem lenne, de nem boldogulok vele. A römi kártyák (52 lapból álló jokerek nélküli francia kártya) egy speciális sorrendjéről van szó, ezt a sorrendet SI STEBBINS sornak nevezik. Lényege: az ábrán látható körben óramutató járásával megegyező irányban a lapok színei periodikusan követik egymást (most pl. pikk, kőr, treff, káró), és bármelyik lap értéke mindig 3-mal több mod 13, mint az előtte levő lap értéke. Előnye ennek a sorrendnek, hogy valójában nincs vége, azaz a kártyacsomag utolsó lapja után következik a kártyacsomag első lapja... A probléma a következő: Két lap színének és értékének ismeretében hogyan lehet gyorsan és könnyen kiszámolni a két lap távolságát. Egyszerű képletet keresek. Mindenki segítségét előre is köszönöm.

[2304] marcius82023-07-19 18:49:50

az előző hozzászólásomhoz ábrát csatolok, hátha így érthetőbb lesz a Si Stebbins sorrend

[2305] nadorp2023-08-09 15:15:34

Írjuk fel a kör kerületére az 1,2...,52 számokat és tegyük a számok mellé a kártyákat a sor definíciója szerint úgy, hogy az 1 mellett legyen a pikk ász.

Ha \(\displaystyle 1\leq i\leq j\leq52\), akkor az \(\displaystyle i\)-dik és \(\displaystyle j\)-dik lap távolsága legyen \(\displaystyle d=\min(j-i,52+i-j)\) ( a nem hosszabb íven levő intervallumot vesszük)

Minden lapot egyértelműen meghatároz egy \(\displaystyle (a,b)\) számpár (\(\displaystyle 0\leq a\leq3,0\leq b\leq12\)), ahol a="szín", b="érték-1" (Most a színekre pikk=0,kőr=1,treff=2,káró=3)

Ekkor a pikk ász a \(\displaystyle (0,0)\) és ha az \(\displaystyle n\)-dik lap az \(\displaystyle (a,b)\), akkor:

\(\displaystyle n-1\equiv a\mod(4)\)

\(\displaystyle 3(n-1)\equiv b\mod(13)\)

Ismert, hogy adott \(\displaystyle (a,b)\) esetén a fenti kongruencia rendszernek egyértelmű megoldása van \(\displaystyle n\)-re \(\displaystyle \mod(52)\), és különböző \(\displaystyle (a,b)\) számpárra - melyek megfelelnek a fenti feltételnek - különböző \(\displaystyle n\)-et kapunk.

Ha most adott két lap \(\displaystyle (a_1,a_2)\) és \(\displaystyle (b_1,b_2)\), melyek az \(\displaystyle n_1\)-dik illetve \(\displaystyle n_2\)-dik helyen vannak, akkor a fentiek szerint

\(\displaystyle n_1-n_2\equiv a_1-a_2\mod(4)\)

\(\displaystyle 3(n_1-n_2)\equiv b_1-b_2\mod(13)\)

——

\(\displaystyle 13(n_1-n_2)\equiv 13(a_1-a_2)\mod(52)\)

\(\displaystyle 12(n_1-n_2)\equiv 4(b_1-b_2)\mod(52)\)

——

\(\displaystyle n_1-n_2\equiv 13(a_1-a_2)-4(b_1-b_2)\mod(52)\)

Innen már látszik a módszer a távolság kiszámítására:

Kiszámoljuk \(\displaystyle 13(a_1-a_2)-4(b_1-b_2)\) értékét és vesszük az 52-es nemnegatív maradékát. Ha ez \(\displaystyle d\) , akkor a távolság \(\displaystyle \min(d,52-d)\)

Előzmény: [2303] marcius8, 2023-07-19 18:45:26
[2306] hanyforint2023-08-16 19:11:59

Találtam egy angol nyelvű pdf-et erről, ami több oldalon keresztül elmagyarázza ezt, itt: "https://web.northeastern.edu/seigen/11Magic/StebbinsStack/Si Stebbins Stack - from Numericana dot com.pdf"

Előzmény: [2303] marcius8, 2023-07-19 18:45:26
[2307] sakkmath2023-11-08 16:50:39

Az N-nel jelölt konvex, n-oldalú sokszöget háromszögekre bontottuk azon átlóival, amelyek nem metszik egymást N belsejében. Hányféleképpen tehetjük meg ezt?

[2308] Róbert Gida2023-11-11 05:36:13

Mármint a teljes particionálás, \(\displaystyle n-2\) darab háromszögre kérdezed? Mert akkor ez ismert, \(\displaystyle C_{n-2}\), az \(\displaystyle n-2\)-edik Catalan szám.

Előzmény: [2307] sakkmath, 2023-11-08 16:50:39
[2309] sakkmath2023-11-11 12:08:44

Igen, és köszönöm a megoldást.

Előzmény: [2308] Róbert Gida, 2023-11-11 05:36:13
[2310] marcius82023-12-09 19:27:11

Adott "n" pont egy kör kerületén. Páronként összekötjök a pontokat. Legfeljebb hány részre osztják ezek az összekötő szakaszok a körlemezt. Előre is köszönök minden segítséget.

[2311] K Robi2023-12-16 12:04:48

\(\displaystyle AD=DQ=DC\\ QC=QB\\ PB=CB\\\)

Sziasztok! Nyilván \(\displaystyle \alpha=82^{\circ}\) , de hogyan tovább? Nem jövök rá...

\(\displaystyle \beta=?\\ \delta=?\\ \varepsilon=?\)

[2312] sakkmath2023-12-16 16:56:42

Ez egy széles körben ismert probléma. A keresett részek száma legyen k. A megoldás:

\(\displaystyle k=\binom{n}{0}+\binom{n}{2}+\binom{n}4=\frac{1}{24}(n^{4}-6n^{3}+23n^{2}-18n+24)\)

Bővebb információk és a feladattal kapcsolatos érdekességek az OEIS-adatbázisban: https://oeis.org/A000127/internal. E linkről kiindulva juthatunk el egy angol nyelvű megoldáshoz: https://maa.org/sites/default/files/Marc_Noy46792.pdf.

Egy részletes, magyar nyelvű megoldást olvashatunk a MATEMATIKAI PROBLÉMAKALAUZ I. című jegyzeben (4.75. probléma*). A kiadványt megvehetjük e-könyves, vagy nyomtatott formátumban is. Érdemes, most kedvezménnyel adják: https://interkonyv.hu/konyvek/kosztolanyi-jozsef-makay-geza-pinter-klara-pinter-lajos-matematikai-problemakalauz-i/

A nyomtatott jegyzet fedőlapjának fotója:

Előzmény: [2310] marcius8, 2023-12-09 19:27:11
[2313] Lpont2023-12-16 17:34:37

Csal az ábra, ha a peremfeltételek igazak, akkor D, P, B nem lehetnek kollineárisak. Némi szögszámolás után P egybeesik Q-val, QCB háromszög szabályos és a kérdőjeles szögek rendre 0, 98, 60.

Előzmény: [2311] K Robi, 2023-12-16 12:04:48
[2314] K Robi2023-12-16 20:07:42

Köszönöm a választ! "Az ábra csak tájékoztató jellegű vázlat, nem pontos méretű." Lehet, hogy hiba volt ezt lehagynom!

Ez kezd érdekes lenni, képzeld, erre a feladatra egy nyolcadikos felvételi előkészítő példatárban bukkantam, és bevallom, nem ment. Pedig nem nyolcadikos vagyok (nagyon nem), és nem is felvételi körülmények között próbálkoztam vele,és nem is pár percem volt rá, ahogy a nyolcadikus felvételizőknek.

Gyanús, hogy itt valami elírás/nyomdahiba lehet? Persze nekem nagyon tetszene, ha te így is megoldod, csaknem tudom követni, részletezed a megoldásod?

Ideteszem a teljes feladatot kivágva meg a megoldást is a példatár szerint.

Előzmény: [2313] Lpont, 2023-12-16 17:34:37
[2315] Lpont2023-12-16 21:45:23

A QCB és a PCB háromszögek hasonlóak, mert mindkettő egyenlő szárú és az alapon fekvő szögeik megegyeznek, ami a C csúcsnál fekvő szög mindkét háromszögben. Ekkor a szárszögek is egyenlőek, tehát CQB szög = PBC szög = ε.

Ha D, P, B pontok kollineárisak, akkor DBC szög = ε és az általad közölt tankönyvi végeredmény szerint a DBC háromszögben a B és C csúcsoknál fekvő szögek összege 16+164=180, vagyis ebben a háromszögben D-nél 0 fok van, a DBC háromszög szakasszá fajul.

Előzmény: [2314] K Robi, 2023-12-16 20:07:42
[2316] BerkoErzsebet2023-12-16 22:32:57

AB oldalának Q pontja helyett AB oldalának Q felezőpontja

Én is foglalkoztam a példával délután. A stílus nagyon ismerős volt. Sejtettem, hogy felvételi feladat. Azt, hogy az ábra tájékoztató jellegű, mindig odaírják.

[2317] Lpont2023-12-16 23:19:06

Miből derül ki, hogy Q felezőpont?

Elvárjuk a 14 éves diáktól, hogy javítsa a kitűzést úgy, hogy megoldható legyen a feladat?

Amennyiben az ábra csak "tájékoztató" jellegű, akkor legyen P a QC szakasz tetszőleges belső pontja, de ne illeszkedjen DB-re, minden más feltétel változatlan.

Ekkor, 8<ε<60 esetén végtelen sok megoldás van.

Előzmény: [2316] BerkoErzsebet, 2023-12-16 22:32:57
[2318] BerkoErzsebet2023-12-16 23:32:59

K Robi írta, hogy példatári feladat. A felvételin remélhetőleg olyan példát adtak, ami néhány perc alatt megoldható. A példatárba valószínű hiányosan, hibásan került bele a példa.

Előzmény: [2317] Lpont, 2023-12-16 23:19:06
[2319] Lpont2023-12-16 23:55:23

Nyilván igazad van, hiszen ez a példa a kiegészítéseddel könnyen oldható, de képzeld el, ha így kerül bele egy felvételi feladatsorba. Mennyi időt el lehet vele feleslegesen tölteni és csak körbe-körbe jár mindig azonosságra jutva.

Előzmény: [2318] BerkoErzsebet, 2023-12-16 23:32:59
[2320] Lpont2023-12-17 00:29:59

Javítom magam......

Ha Q felezi AB-t, akkor sem jó az ábra és nem oldható meg a feladat, mert DBC háromszögben D-nél 0 fok kell legyen, ha a kiindulási feltételek teljesülnek.

Előzmény: [2319] Lpont, 2023-12-16 23:55:23
[2321] Lpont2023-12-17 08:49:00

Tűzzük ki nehezített változatban a feladatot.

Létezik-e olyan nem elfajuló konvex QBCD négyszög melynek Q-nál és D-nél levő szögei rendre 98 és 16fok, továbbá QD=DC, CQ=QB és a keresett négyszög átlóinak P metszéspontjára teljesül a PB=BC feltétel.

a) Ha van ilyen négyszög, akkor határozzuk meg a hiányzó szögeit.

b) Indokoljuk, ha nincs ilyen tulajdonságú négyszög.

Előzmény: [2316] BerkoErzsebet, 2023-12-16 22:32:57
[2322] erwinsmith2023-12-27 13:47:18

köszönöm

[2323] marcius82024-01-11 20:45:17

Eredeti megfogalmazásomban a következőképpen nézett ki az előbb felvetett probléma, amelyet Kós Géza Tanár Úr átfogalmazott, és a KöMaL-pontversenybe került be: (érdekes matek feladatok, [3668], Kömal, 2013 március, "A" feladatok)

Egy játékot találtam ki. Mivel egy játékkal akkor foglalkoznak sokan, ha szabályai egyszerűek, ugyanakkor nem könnyen végigjátszható, ezért a következő játékot találtam ki:

A magyar kártyacsomag összetétele: A magyar kártyacsomag 32 lapot tartalmaz. Minden lapnak van színe és értéke. A színek lehetnek: „piros” (tavasz), „tök” (nyár), „zöld” (ősz), „makk” (tél). Az értékek lehetnek: „VII”, „VIII”, „IX”, „X”, „alsó”, „felső”, „király”, „ász”. A kártyacsomagban minden lehetséges szín-érték párosítás előfordul.

A nyolc sorból és négy oszlopból álló táblázatban elhelyezett magyar kártyacsomag lapjai akkor vannak rendezett sorrendben, ha a következő feltételek teljesülnek:

• A táblázat minden oszlopában található négy lap színének sorrendje felülről lefelé haladva: „piros” (tavasz), „tök” (nyár), „zöld” (ősz), „makk” (tél).

• A táblázat minden sorában található nyolc lap értékének sorrendje balról jobbra haladva: „VII”, „VIII”, „IX”, „X”, „alsó”, „felső”, „király”, „ász”.

A játék szabálya: A kártyacsomag lapjai véletlenszerű sorrendben egy négy sorból és nyolc oszlopból álló táblázatban vannak elhelyezve. A játék során egyszerre mindig két lapot lehet megcserélni. Két lapot csak akkor lehet megcserélni, ha a két lap ugyanabban a sorban vagy ugyanabban az oszlopban van, továbbá (és) ha a két lap színe vagy értéke ugyanaz. A játék célja, hogy a kártyalapok sorrendje rendezett legyen.

Az érdekes az hogy a véletlenszerűen összekevert állapotból majdnem mindig abba az állapotba jutok, hogy a rendezett állapothoz képest a "makk ász" és a "tök király" fel van cserélve. Sajnos, nem tudok rájönni, hogy ez az én ügyetlenségem (gyanítom, hogy igen), de ugyanakkor nem hiszem hogy ennek törvényszerűen így kell lenni. Arra kérek bárkit, hogy ha ezt a jelenséget meg tudja indokolni, vagy meg tudja oldani (tehát ha a rendezett állapothoz képest csak a "makk ász" és a "tök király" van felcserélve, akkor ez az állapot rendezhető vagy sem), írjon a zoltanbertalan680308@gmail.com címre.

[2324] marcius82024-01-11 21:50:01

Érdekelne az is, hogy rendezett állapotból kiindulva, megengedett cseréket végrehajtva, a kártyalapok milyen permutációja érhető el. Előre is köszönöm mindenki segítségét.

Előzmény: [2323] marcius8, 2024-01-11 20:45:17
[2326] marcius82024-02-19 14:29:54

Parabola alatti területet hogyan lehet meghatározni teljesen elemi módszerekkel, azaz az eljárásban legyen benne végtelen sor határértéke, és ne legyen integrálszámítás? Előre is köszönök minden segítséget.

[2327] Fálesz Mihály2024-02-19 20:05:42

A parabola alatti terület kb ugyanaz, mint a gúla térfogata.

A kocka felbontható három egybevágó ferde gúlára.

Előzmény: [2326] marcius8, 2024-02-19 14:29:54
[2328] SmallPotato2024-02-20 02:15:51

Ebben a szakdolgozatban (és egyéb helyeken, "téglányösszeg" jeligére) szerintem találsz választ.

Gyanús ugyan, hogy "az eljárásban legyen benne végtelen sor határértéke" kitételedből nem maradt-e ki egy "ne", de kimondott vagy kimondatlan határérték nélkül szerintem a dolog nem megoldható.

Előzmény: [2326] marcius8, 2024-02-19 14:29:54
[2329] marcius82024-02-20 05:03:35

Köszönöm az eddigi segítségeket. A legutóbbi hsz-omból kimaradt egy "ne". Tehát javítva: Hogyan lehet meghatározni egy parabola alatti területet, hogy az eljárásban NE legyen végtelen sor és NE legyen határérték, és NE legyen differenciálszámítás vagy integrálszámítás, mert hogy ezek nem elemi módszerek.

Előzmény: [2326] marcius8, 2024-02-19 14:29:54
[2330] Sinobi2024-02-21 14:54:14

És a kör területét?

[2331] SmallPotato2024-02-23 18:08:30

Erre azért tényleg kíváncsi vagyok most már magam is.

(a határérték felhasználása nélküli megoldásokról az a közismert "bizonyítás" jut eszembe, amely szerint a négyzet átlója az oldalak összegével egyenlő)

Előzmény: [2329] marcius8, 2024-02-20 05:03:35

  [1]    [2]    [3]    [4]    [5]    [6]    [7]    [8]    [9]    [10]    [11]    [12]    [13]    [14]    [15]    [16]    [17]    [18]    [19]    [20]    [21]    [22]    [23]    [24]    [25]    [26]    [27]    [28]    [29]    [30]    [31]    [32]    [33]    [34]    [35]    [36]    [37]    [38]    [39]    [40]    [41]    [42]    [43]    [44]    [45]    [46]